K
Khách

Hãy nhập câu hỏi của bạn vào đây, nếu là tài khoản VIP, bạn sẽ được ưu tiên trả lời.

1 tháng 1 2021

giả sử \(a\ge b\ge c\ge0\)

Ta có: \(a+\frac{b}{2}-\frac{a^2+ab+b^2}{a+b}=\frac{1}{2}\left(ab-b^2\right)\ge0\Rightarrow a+\frac{b}{2}\ge\frac{a^2+ab+b^2}{a+b}\)

\(b+\frac{a}{2}-\frac{a^2+ab+b^2}{a+b}=\frac{1}{2}\left(ab-a^2\right)\le0\Rightarrow b+\frac{a}{2}\le\frac{a^2+ab+b^2}{a+b}\)

Tương tự: \(b+\frac{c}{2}\ge\frac{b^2+bc+c^2}{b+c}\ge c+\frac{b}{2};a+\frac{c}{2}\ge\frac{a^2+ac+c^2}{a+c}\ge c+\frac{a}{2}\)

Lại có:+) \(\frac{a^3-b^3}{a+b}+\frac{b^3-c^3}{b+c}+\frac{c^3-a^3}{c+a}\)

\(=\left(a-b\right)\frac{a^2+ab+b^2}{a+b}+\left(b-c\right)\frac{b^2+bc+c^2}{b+c}-\left(a-c\right)\frac{a^2+ac+c^2}{a+c}\)

\(\ge\left(a-b\right)\left(b+\frac{a}{2}\right)+\left(b-c\right)\left(c+\frac{a}{2}\right)-\left(a-c\right)\left(a+\frac{c}{2}\right)\)

\(\ge\frac{-1}{4}\left[\left(a-b\right)^2+\left(b-c\right)^2+\left(c-a\right)^2\right]\left(1\right)\)

+) \(\frac{a^3-b^3}{a+b}+\frac{b^3-c^3}{b+c}+\frac{c^3-a^3}{c+a}\)

\(=\left(a-b\right)\frac{a^2+ab+b^2}{a+b}+\left(b-c\right)\frac{b^2+bc+c^2}{b+c}-\left(a-c\right)\frac{a^2+ac+c^2}{a+c}\)

\(\le\left(a-b\right)\left(a+\frac{b}{2}\right)+\left(b-c\right)\left(b+\frac{c}{2}\right)-\left(a-c\right)\left(c+\frac{a}{2}\right)\)

\(\le\frac{1}{4}\left[\left(a-b\right)^2+\left(b-c\right)^2+\left(c-a\right)^2\right]\left(2\right)\)

Từ 1,2 => đpcm

2 tháng 1 2021

BĐT đã cho tuong duong voi:

\(\left|\frac{\left(a-b\right)\left(b-c\right)\left(c-a\right)\left(ab+bc+ca\right)}{\left(a+b\right)\left(b+c\right)\left(c+a\right)}\right|\le\frac{1}{4}\left[\Sigma\left(a-b\right)^2\right]\)

Theo AM-GM ta có: \(\left(ab+bc+ca\right)\le\frac{9}{8}\cdot\frac{\left(a+b\right)\left(b+c\right)\left(c+a\right)}{a+b+c}\)

Có: \(VT\le\frac{9}{8}\left|\frac{\sqrt{\left(a-b\right)^2\left(b-c\right)^2\left(c-a\right)^2}}{\left(a+b+c\right)}\right|=\frac{9\sqrt{\left(a-b\right)^2\left(b-c\right)^2\left(c-a\right)^2}}{8\left(a+b+c\right)}\)

Cần chứng minh: \(4\left(a+b+c\right)^2\left[\Sigma\left(a-b\right)^2\right]^2\ge9\left(a-b\right)^2\left(b-c\right)^2\left(c-a\right)^2\)

Rõ ràng \(\Sigma\left(a-b\right)^2\ge3\sqrt[3]{\left(a-b\right)^2\left(b-c\right)^2\left(c-a\right)^2}\)

Cần cm: \(36\left(a+b+c\right)^2\sqrt[3]{\left(a-b\right)^4\left(b-c\right)^4\left(c-a\right)^4}\ge9\sqrt[3]{\left(a-b\right)^6\left(b-c\right)^6\left(c-a\right)^6}\)

Hay \(4\left(a+b+c\right)^2\ge\sqrt[3]{\left(a-b\right)^2\left(b-c\right)^2\left(c-a\right)^2}\)

Tiếp tục là điều hiển nhiên do \(VT\ge4\left[\left(a+b+c\right)^2-3\left(ab+bc+ca\right)\right]\)

\(=2\left[\left(a-b\right)^2+\left(b-c\right)^2+\left(c-a\right)^2\right]\)

\(\ge6\sqrt[3]{\left(a-b\right)^2\left(b-c\right)^2\left(c-a\right)^2}\ge VP\)

Đẳng thức xảy ra khi \(\hept{\begin{cases}\left(a-b\right)\left(b-c\right)\left(c-a\right)=0\\a-b=b-c=c-a\\a=b=c\end{cases}}\Leftrightarrow a=b=c.\)

28 tháng 8 2020

Áp dụng giả thiết và một đánh giá quen thuộc, ta được: \(16\left(a+b+c\right)\ge\frac{1}{a}+\frac{1}{b}+\frac{1}{c}=\frac{ab+bc+ca}{abc}=\frac{\left(ab+bc+ca\right)^2}{abc\left(ab+bc+ca\right)}\ge\frac{3\left(a+b+c\right)}{ab+bc+ca}\)hay \(\frac{1}{6\left(ab+bc+ca\right)}\le\frac{8}{9}\)

Đến đây, ta cần chứng minh \(\frac{1}{\left(a+b+\sqrt{2\left(a+c\right)}\right)^3}+\frac{1}{\left(b+c+\sqrt{2\left(b+a\right)}\right)^3}+\frac{1}{\left(c+a+\sqrt{2\left(c+b\right)}\right)^3}\le\frac{1}{6\left(ab+bc+ca\right)}\)

 Áp dụng bất đẳng thức Cauchy cho ba số dương ta có \(a+b+\sqrt{2\left(a+c\right)}=a+b+\sqrt{\frac{a+c}{2}}+\sqrt{\frac{a+c}{2}}\ge3\sqrt[3]{\frac{\left(a+b\right)\left(a+c\right)}{2}}\)hay \(\left(a+b+\sqrt{2\left(a+c\right)}\right)^3\ge\frac{27\left(a+b\right)\left(a+c\right)}{2}\Leftrightarrow\frac{1}{\left(a+b+2\sqrt{a+c}\right)^3}\le\frac{2}{27\left(a+b\right)\left(a+c\right)}\)

Hoàn toàn tương tự ta có \(\frac{1}{\left(b+c+2\sqrt{b+a}\right)^3}\le\frac{2}{27\left(b+c\right)\left(b+a\right)}\)\(\frac{1}{\left(c+a+2\sqrt{c+b}\right)^3}\le\frac{2}{27\left(c+a\right)\left(c+b\right)}\)

Cộng theo vế các bất đẳng thức trên ta được \(\frac{1}{\left(a+b+\sqrt{2\left(a+c\right)}\right)^3}+\frac{1}{\left(b+c+\sqrt{2\left(b+a\right)}\right)^3}+\frac{1}{\left(c+a+\sqrt{2\left(c+b\right)}\right)^3}\le\frac{4\left(a+b+c\right)}{27\left(a+b\right)\left(b+c\right)\left(c+a\right)}\)Phép chứng minh sẽ hoàn tất nếu ta chỉ ra được \(\frac{4\left(a+b+c\right)}{27\left(a+b\right)\left(b+c\right)\left(c+a\right)}\le\frac{1}{6\left(ab+bc+ca\right)}\)\(\Leftrightarrow\left(a+b\right)\left(b+c\right)\left(c+a\right)\ge\frac{8}{9}\left(ab+bc+ca\right)\left(a+b+c\right)\)

Đây là một đánh giá đúng, thật vậy: đặt a + b + c = p; ab + bc + ca = q; abc = r thì bất đẳng thức trên trở thành \(pq-r\ge\frac{8}{9}pq\Leftrightarrow\frac{1}{9}pq\ge r\)*đúng vì \(a+b+c\ge3\sqrt[3]{abc}\)\(ab+bc+ca\ge3\sqrt[3]{\left(abc\right)^2}\))

Vậy bất đẳng thức được chứng minh

Đẳng thức xảy ra khi \(a=b=c=\frac{1}{4}\)

7 tháng 5 2020

\(\Leftrightarrow\Sigma\sqrt{\frac{3a^3}{\left[5a^2+\left(b+c\right)^2\right]\left(a+b+c\right)}}\le1\)

Theo Am-GM: \(VT=\Sigma\sqrt{\frac{3a^2}{5a^2+\left(b+c\right)^2}.\frac{a}{a+b+c}}\le\Sigma\frac{3a^2}{2\left(5a^2+\left(b+c\right)^2\right)}+\frac{1}{2}\)

Như vậy nó là đủ để chứng minh rằng: \(\Sigma\frac{3a^2}{5a^2+\left(b+c\right)^2}\le1\)

Giả sử \(c=min\left\{a,b,c\right\}\) nó tương đương:

$$2\, \left( a-b \right) ^{2} \left( 3\,c+a+b \right) \left( -c+a+b
 \right) \left( {a}^{2}+2\,ab+{b}^{2}+5\,{c}^{2} \right) +2\,c
 \left( a-c \right) \left( b-c \right) \left( 3\,{a}^{3}+9\,{a}^{2}b
+17\,c{a}^{2}+9\,a{b}^{2}-20\,abc+3\,{c}^{2}a+3\,{b}^{3}+17\,c{b}^{2}+
3\,{c}^{2}b+{c}^{3} \right) \geqq 0$$

(Gõ Latex, không hiện thì vô thống kê hỏi đáp xem)

Đây là điều hiển nhiên/

PS: Bài này quan trọng là ý tưởng phá căn thôi chứ không có gì khó. Lúc đầu UCT bất đẳng thức cuối cho đẹp nhưng phải xét các TH mệt lắm, chưa rành nên không làm cách đó:D

7 tháng 5 2020

Chứng minh: \(\Sigma\frac{3a^2}{5a^2+\left(b+c\right)^2}\le1\), cách 2:

Đổi biến sang pqr: (Vô thống kê hỏi đáp xem nếu olm không hiện Latex)

Nếu \(p^2\le4q\) ta cần:

$$2/9\,p \left( 19\,{p}^{2}-36\,q \right) \left( {p}^{3}-4\,qp+9\,r
 \right) -4/9\, \left( {p}^{2}-3\,q \right) \left( {p}^{2}-4\,q
 \right) \left( 5\,{p}^{2}-3\,q \right) \geqq 0$$

(Hiển nhiên)

Nếu \(p^2\ge4q\) thì cần chứng minh:

$$2\,p \left( 19\,{p}^{2}-36\,q \right) r+2\, \left( {p}^{2}-4\,q
 \right) \left( {p}^{4}-2\,{q}^{2} \right) \geqq 0$$

(Hiển nhiên)

Từ 2 TH trên ta thu được điều phải chứng minh.

Giúp mình với! Mình đang cần gấp. Các bạn làm được bài nào thì giúp đỡ mình nhé! Cảm ơn!Bài 1: Cho các số thực dương a,b,c. Chứng minh rằng:\(\frac{a^2}{\sqrt{\left(2a^2+b^2\right)\left(2a^2+c^2\right)}}+\frac{b^2}{\sqrt{\left(2b^2+c^2\right)\left(2b^2+a^2\right)}}+\frac{c^2}{\sqrt{\left(2c^2+a^2\right)\left(2c^2+b^2\right)}}\le1\).Bài 2: Cho các số thực dương a,b,c,d. Chứng minh...
Đọc tiếp

Giúp mình với! Mình đang cần gấp. Các bạn làm được bài nào thì giúp đỡ mình nhé! Cảm ơn!

Bài 1: Cho các số thực dương a,b,c. Chứng minh rằng:

\(\frac{a^2}{\sqrt{\left(2a^2+b^2\right)\left(2a^2+c^2\right)}}+\frac{b^2}{\sqrt{\left(2b^2+c^2\right)\left(2b^2+a^2\right)}}+\frac{c^2}{\sqrt{\left(2c^2+a^2\right)\left(2c^2+b^2\right)}}\le1\).

Bài 2: Cho các số thực dương a,b,c,d. Chứng minh rằng:

\(\frac{a-b}{a+2b+c}+\frac{b-c}{b+2c+d}+\frac{c-d}{c+2d+a}+\frac{d-a}{d+2a+b}\ge0\).

Bài 3: Cho các số thực dương a,b,c. Chứng minh rằng:

\(\frac{\sqrt{b+c}}{a}+\frac{\sqrt{c+a}}{b}+\frac{\sqrt{a+b}}{c}\ge\frac{4\left(a+b+c\right)}{\sqrt{\left(a+b\right)\left(b+c\right)\left(c+a\right)}}\).

Bài 4:Cho a,b,c>0, a+b+c=3. Chứng minh rằng: 

a)\(\frac{a^3}{a^2+ab+b^2}+\frac{b^3}{b^2+bc+c^2}+\frac{c^3}{c^2+ca+a^2}\ge1\).

b)\(\frac{a^3}{a^2+b^2}+\frac{b^3}{b^2+c^2}+\frac{c^3}{c^2+a^2}\ge\frac{3}{2}\).

c)\(\frac{a+1}{b^2+1}+\frac{b+1}{c^2+1}+\frac{c+1}{a^2+1}\ge3\).

Bài 5: Cho a,b,c >0. Chứng minh rằng:

\(\frac{2a^2+ab}{\left(b+c+\sqrt{ca}\right)^2}+\frac{2b^2+bc}{\left(c+a+\sqrt{ab}\right)^2}+\frac{2c^2+ca}{\left(a+b+\sqrt{bc}\right)^2}\ge1\).

8
21 tháng 10 2019

1) Áp dụng bunhiacopxki ta được \(\sqrt{\left(2a^2+b^2\right)\left(2a^2+c^2\right)}\ge\sqrt{\left(2a^2+bc\right)^2}=2a^2+bc\), tương tự với các mẫu ta được vế trái \(\le\frac{a^2}{2a^2+bc}+\frac{b^2}{2b^2+ac}+\frac{c^2}{2c^2+ab}\le1< =>\)\(1-\frac{bc}{2a^2+bc}+1-\frac{ac}{2b^2+ac}+1-\frac{ab}{2c^2+ab}\le2< =>\)

\(\frac{bc}{2a^2+bc}+\frac{ac}{2b^2+ac}+\frac{ab}{2c^2+ab}\ge1\)<=> \(\frac{b^2c^2}{2a^2bc+b^2c^2}+\frac{a^2c^2}{2b^2ac+a^2c^2}+\frac{a^2b^2}{2c^2ab+a^2b^2}\ge1\)  (1) 

áp dụng (x2 +y2 +z2)(m2+n2+p2\(\ge\left(xm+yn+zp\right)^2\)

(2a2bc +b2c2 + 2b2ac+a2c2 + 2c2ab+a2b2). VT\(\ge\left(bc+ca+ab\right)^2\)   <=> (ab+bc+ca)2. VT \(\ge\left(ab+bc+ca\right)^2< =>VT\ge1\)  ( vậy (1) đúng)

dấu '=' khi a=b=c

21 tháng 10 2019

4b, \(\frac{a^3}{a^2+b^2}+\frac{b^3}{b^2+c^2}+\frac{c^3}{c^2+a^2}=1-\frac{ab^2}{a^2+b^2}+1-\frac{bc^2}{b^2+c^2}+1-\frac{ca^2}{a^2+c^2}\)

\(\ge3-\frac{ab^2}{2ab}-\frac{bc^2}{2bc}-\frac{ca^2}{2ac}=3-\frac{\left(a+b+c\right)}{2}=\frac{3}{2}\)

8 tháng 7 2016

bài 2 thì bạn áp dụng bdt cô si với lựa chọn điểm rơi  hoặc bdt holder  ( nó giống kiểu bunhia ngược ) . bai 1 thi ap dung cai nay \(\frac{1}{x}+\frac{1}{y}>=\frac{1}{x+y}\)  câu 1 khó hơn nhưng bạn biết lựa chọn điểm rơi với áp dụng bdt phụ kia là ok .

9 tháng 7 2016

Bài 1:Đặt VT=A

Dùng BĐT \(\left(x+y+z\right)\left(\frac{1}{x}+\frac{1}{y}+\frac{1}{z}\right)\ge9\Rightarrow\frac{1}{x+y+z}\le\frac{1}{9}\left(\frac{1}{x}+\frac{1}{y}+\frac{1}{z}\right)x,y,z>0\)

Áp dụng vào bài toán trên với x=a+c;y=b+a;z=2b ta có:

\(\frac{ab}{a+3b+2c}=\frac{ab}{\left(a+c\right)+\left(b+c\right)+2b}\le\frac{ab}{9}\left(\frac{1}{a+c}+\frac{1}{b+c}+\frac{1}{2b}\right)\)

Tương tự với 2 cái còn lại

\(A\le\frac{1}{9}\left(\frac{bc+ac}{a+b}+\frac{bc+ab}{a+c}+\frac{ab+ac}{b+c}\right)+\frac{1}{18}\left(a+b+c\right)\)

\(\Rightarrow A\le\frac{1}{9}\left(a+b+c\right)+\frac{1}{18}\left(a+b+c\right)=\frac{a+b+c}{6}\)

Đẳng thức xảy ra khi a=b=c 

Bài 2:

Biến đổi BPT \(4\left(\frac{a^3}{\left(1+b\right)\left(1+c\right)}+\frac{b^3}{\left(1+c\right)\left(1+a\right)}+\frac{c^3}{\left(1+a\right)\left(1+b\right)}\right)\ge3\)

\(\Rightarrow\frac{a^3}{\left(1+b\right)\left(1+c\right)}+\frac{b^3}{\left(1+c\right)\left(1+a\right)}+\frac{c^3}{\left(1+a\right)\left(1+b\right)}\ge\frac{3}{4}\)

Dự đoán điểm rơi xảy ra khi a=b=c=1

\(\frac{a^3}{\left(1+b\right)\left(1+c\right)}+\frac{1+b}{8}+\frac{1+c}{8}\ge\frac{3a}{4}\)

Tương tự suy ra

\(VT\ge\frac{2\left(a+b+c\right)-3}{4}\ge\frac{2\cdot3\sqrt{abc}-3}{4}=\frac{3}{4}\)

8 tháng 2 2017

Áp dụng C-S

\(\frac{a}{2a+b+c}=\frac{a}{\left(a+b\right)+\left(a+c\right)}\le\frac{a}{4}\left(\frac{1}{a+b}+\frac{1}{a+c}\right)\)

9 tháng 2 2017

b) chính là USAMO 2004. Đây là lời giải cung cấp bởi "http://www.artofproblemsolving.com/wiki/index.php/2004_USAMO_Problems/Problem_5"

Ta chứng minh được \(x^5+1\ge x^3+x^2\) suy ra \(x^5-x^2+3\ge x^3+2\).

Ta chỉ cần CM được \(\left(a^3+1+1\right)\left(1+b^3+1\right)\left(1+1+c^3\right)\ge\left(a+b+c\right)^3\)

Nhưng đây chính là BĐT Holder cho 3 bộ số mỗi bộ 3 số.